¿Alguna base pertenece a un conjunto de funciones propias de algún observable?

En los cursos de Mecánica Cuántica en ocw.mit.edu se dice que cualquier función de onda puede expandirse como una superposición de las funciones propias de cualquier observable, es decir, las funciones propias forman una base propia para nuestro espacio.

Sin embargo, si tomamos una base arbitraria para nuestro espacio, ¿es ese espacio una base propia correspondiente a un observable? En otras palabras, ¿es cualquier base un conjunto de funciones propias de algún observable?

Respuestas (3)

¿Es cualquier base un conjunto de funciones propias de algún observable?

Esto depende de qué es exactamente lo que desea contar como 'observable' y lo que desea descartar.


El primer ejemplo que debe considerar es simplemente el operador de identidad I , para el cual cualquier vector distinto de cero es un vector propio con valor propio 1 . (Físicamente, esto corresponde al número 1 entonces, en el mismo sentido que los observables en la mecánica clásica son funciones F ( q , pag ) de posición y momento, el operador identidad corresponde a la función F ( q , pag ) 1 .) Entonces, la respuesta aquí es , aunque eso podría requerir lo que algunas personas consideran un observable 'trivial'.


En respuesta a eso, el próximo paso tentador es modificar la solicitud para

¿Es cualquier base un conjunto de funciones propias de algún observable no constante?

que descarta el operador identidad y sus múltiplos.

Una vez que haces eso, la respuesta es no . De la forma en que lo expresó, el conjunto de bases no necesita ser ortogonal, y si la base no es ortogonal, entonces no es posible construir un operador hermitiano con esa base como sus funciones propias, ya que cualquier base propia de un operador hermitiano debe ser ortogonal .

(Eso no es del todo cierto, como sucede, porque puede tener vectores de base no ortogonales dentro de cualquier espacio propio degenerado, pero eso nos devuelve al caso anterior: el observable simplemente actúa como la identidad dentro de ese subespacio. En términos de la pregunta, significa que si la base β no es ortogonal pero contiene dos subconjuntos β 1 = { tu i } y β 2 = { v j } que son mutuamente ortogonales, entonces puedes construir un observable no constante A tal que A tu i = tu tu i y A v j = v v j para dos numeros fijos tu y v . Una vez más, sin embargo, eso es básicamente una evasión trivial, y no trae nada nuevo que el operador de identidad no haya hecho ya).


Por lo tanto, v2 de la declaración debe corregirse:

¿Es cualquier base ortogonal un conjunto de funciones propias de algún observable no constante?

Aquí la respuesta es : dejar β = { v norte : norte = 1 , 2 , } sea ​​una base ortonormal contable. (Y sí, debe ser contable si desea tener alguna oportunidad de una solución viable). Entonces existe un operador autoadjunto lineal único A que se extiende

A v norte = 1 norte v norte
a todo el espacio de Hilbert, y ese operador tiene β como su base propia única sin degeneraciones en ninguna parte.


Sin embargo, la solución anterior producirá un operador autoadjunto que (aunque está perfectamente bien definido matemáticamente) puede ser completamente inaccesible para cualquier experimento físico imaginable en su sistema, por lo que nuevamente puede verse como no particularmente satisfactorio. La solución a eso es introducir el concepto de un observable físicamente accesible, que se construye comenzando con un conjunto básico (digamos, X ^ y pag ^ ) y luego permitir cualquier combinación lineal, potencias de operadores existentes y conmutadores de operadores existentes. (Esto representa un cambio de pensamiento, que está bien ejemplificado en el cambio entre §2.3.1.b y §3.2.1.a de arXiv:1211.5627 ).

Entonces, con eso, puedes preguntar

¿Es cualquier base ortogonal un conjunto de funciones propias de algún observable físicamente accesible no constante?

y aquí la respuesta es no , hay algunos sistemas perfectamente razonables con espacios de estado perfectamente razonables y álgebras de observables perfectamente razonables, que permiten bases perfectamente razonables que no son bases propias de ningún observable en el álgebra elegida. (Desafortunadamente, sin embargo, no tengo ningún ejemplo concreto a mano. Pero dentro de esa definición estrecha, es una buena pregunta de seguimiento si desea publicarla).


Entonces, ¿cuál es la respuesta? Sí. Y no. Dependiendo de lo que entienda por "base" y por "observable".

Muy buena respuesta, Emilio. Sin embargo, tengo curiosidad por algo: en el segundo caso que discute, presenta un conjunto básico β , y afirmas que la cardinalidad del conjunto debe ser contable para tener una solución viable. Sé que en principio podemos construir espacios vectoriales con bases incontables, matemáticamente no hay problema ahí, pero ¿su punto es que tal base incontable es impenetrable para las herramientas de un físico teórico?
@N.Steinle No. Pero si tiene una base incontable, entonces (i) su espacio de Hilbert no es separable, en cuyo caso está muy alejado de lo que generalmente maneja QM, y en realidad no está haciendo cualquier cosa que sea físicamente aplicable en absoluto, o (ii) su "base" continua es algo así como los estados de impulso en el QM 'canónico', y los "estados básicos" en realidad no viven en el espacio de Hilbert (y necesitaría un espacio de Hilbert manipulado, también conocido como un triplete de Gelfand, para describirlos rigurosamente) y está muy lejos de lo que puede manejar la sofisticación matemática mostrada por OP.
(mientras que, al mismo tiempo, en realidad no cambia el contenido de la respuesta en absoluto; los resultados son exactamente los mismos, solo es más complicado probarlos correctamente).
@EmilioPisanty Considere la regla de superselección de la carga eléctrica y un operador autoadjunto que no conmuta con esa carga. No es un observable y su base Hilbert de vectores propios es un posible contraejemplo que estaba buscando. Otra posibilidad es la presencia de un grupo de calibre no abeliano: los operadores autoadjuntos que no conmutan con el representante del grupo de calibre no son observables...
@ValterMoretti No entiendo lo que intentas decir. El punto principal que estoy tratando de hacer aquí es que hay versiones matemáticamente bien definidas de la pregunta que tienen una respuesta positiva, pero que a veces los 'observables' requeridos por la base elegida pueden no ser completamente satisfactorios, a pesar de ser perfectamente -operadores hermitianos legítimos. Si entendí su comentario correctamente, ¿solo está proporcionando más formas para que los operadores hermitianos sean insatisfactorios como observables?
Solo quería señalar algunas bases de Hilbert que no son bases propias de ningún observable, pero tal vez no entendí bien este punto en su respuesta.
@Valter De hecho, estaba buscando contraejemplos, pero suenan demasiado complicados. Creo que debería ser posible hacer esto en vanilla 1D QM.
@EmilioPisanty Creo que las que señalé son las únicas posibilidades debido a razones de principios físicos. Las restantes se deben a razones prácticas o tecnológicas... Pero creo que estas últimas te interesan...
@EmilioPisanty El tema es muy delicado ya que tiene consecuencias sobre varios supuestos comunes en QT. Por ejemplo, si no asume que todos los operadores autoadjuntos representan observables, generalmente es falso que los sistemas compuestos se describan en términos de productos tensoriales.

En otras palabras, ¿es cualquier base un conjunto de funciones propias de algún observable?

EN general, no, ya que requerimos que los observables físicos tengan ciertas propiedades. Sin embargo, matemáticamente hablando, una función puede expresarse en cualquier base ortogonal, pero eso no significa que la base sea físicamente significativa.

Una función de onda se puede escribir como una combinación lineal de cualquier base bien definida (es decir, es ortonormal) y, por supuesto, en principio se puede definir un operador que corresponda a esa base. Sin embargo , este operador no corresponde necesariamente a un observable físico. Por ejemplo, en el ejemplo del potencial del oscilador armónico, podemos expresar la función de onda del sistema como una superposición de estados coherentes, pero el operador que definiría a partir de los estados coherentes (un operador ascendente o descendente) no representa un observable físico.

Entonces, formalmente hablando, requerimos que los observables físicos sean hermitianos, de modo que su base correspondiente sea ortonormal en algún intervalo y sus valores propios tengan valores reales (por lo que proporcionan valores esperados de valores reales). Este es un axioma de la teoría de la mecánica cuántica.

Probablemente no, pero puede depender de lo que entiendas por observable.

Cualquier operador hermitiano tendrá un conjunto básico completo de funciones propias. Se ha encontrado que algunos operadores hermitianos están asociados con observables. Se requiere que los operadores que representan observables sean hermitianos porque deben tener valores esperados reales.

Cualquier suma lineal de operadores hermíticos también sería hermítica, y si estuviera compuesta por observables, esa cantidad también podría observarse. No estoy seguro de cómo probar que el conjunto de posibles estados de bases completas es más grande que el conjunto que podría construir a partir de sumas lineales arbitrarias de observables existentes, pero probablemente pueda probarlo.

La pregunta del OP es esencialmente si cada base corresponde a un observable. Si dice que existen operadores hermetianos con autobases ortonormales que no se corresponden con observables, ¿entonces la respuesta a la pregunta no es simplemente "no"?